Find a number having minimum sum of distances between a set of numbers












5












$begingroup$


Lets say we have a set of numbers ${ 5, 7, 1, 2, 5, 100 }$. I want to find a number $x$ such that the sum of distances of every number from the set to $x$ is minimal.



My first thought was that $x$ is the average of all elements of the set: $frac{5+7+1+2+5+100}{6}$, but it is not true, it fails the above example.



Any help or hint will be appriciated, thanks.










share|cite|improve this question











$endgroup$












  • $begingroup$
    The answer is wrong. For example, $100$ is closer to each element than $104$ is.
    $endgroup$
    – Harnak
    yesterday






  • 2




    $begingroup$
    math.stackexchange.com/questions/113270/…
    $endgroup$
    – Matti P.
    yesterday










  • $begingroup$
    @Harnak Sorry but I don't quite understand your statement. $104$ is the total distance $sum_{x in S} |x - n|$ where $n$ is the desired number and $S = {1, 2, 5, 5, 7, 100}$. How do you achieve $100$? See my proof.
    $endgroup$
    – L. F.
    yesterday










  • $begingroup$
    @L.F., I was just answering to the OP before editing. He stated that the solution was 104, but I thought he referred to the minimizer and not to the distance. So, that's why I provided an example of why 104 couldn't be a minimizer. I think I just misinterpreted what he meant.
    $endgroup$
    – Harnak
    yesterday










  • $begingroup$
    @Harnak Oh, never mind. Everybody makes mistakes :)
    $endgroup$
    – L. F.
    yesterday
















5












$begingroup$


Lets say we have a set of numbers ${ 5, 7, 1, 2, 5, 100 }$. I want to find a number $x$ such that the sum of distances of every number from the set to $x$ is minimal.



My first thought was that $x$ is the average of all elements of the set: $frac{5+7+1+2+5+100}{6}$, but it is not true, it fails the above example.



Any help or hint will be appriciated, thanks.










share|cite|improve this question











$endgroup$












  • $begingroup$
    The answer is wrong. For example, $100$ is closer to each element than $104$ is.
    $endgroup$
    – Harnak
    yesterday






  • 2




    $begingroup$
    math.stackexchange.com/questions/113270/…
    $endgroup$
    – Matti P.
    yesterday










  • $begingroup$
    @Harnak Sorry but I don't quite understand your statement. $104$ is the total distance $sum_{x in S} |x - n|$ where $n$ is the desired number and $S = {1, 2, 5, 5, 7, 100}$. How do you achieve $100$? See my proof.
    $endgroup$
    – L. F.
    yesterday










  • $begingroup$
    @L.F., I was just answering to the OP before editing. He stated that the solution was 104, but I thought he referred to the minimizer and not to the distance. So, that's why I provided an example of why 104 couldn't be a minimizer. I think I just misinterpreted what he meant.
    $endgroup$
    – Harnak
    yesterday










  • $begingroup$
    @Harnak Oh, never mind. Everybody makes mistakes :)
    $endgroup$
    – L. F.
    yesterday














5












5








5





$begingroup$


Lets say we have a set of numbers ${ 5, 7, 1, 2, 5, 100 }$. I want to find a number $x$ such that the sum of distances of every number from the set to $x$ is minimal.



My first thought was that $x$ is the average of all elements of the set: $frac{5+7+1+2+5+100}{6}$, but it is not true, it fails the above example.



Any help or hint will be appriciated, thanks.










share|cite|improve this question











$endgroup$




Lets say we have a set of numbers ${ 5, 7, 1, 2, 5, 100 }$. I want to find a number $x$ such that the sum of distances of every number from the set to $x$ is minimal.



My first thought was that $x$ is the average of all elements of the set: $frac{5+7+1+2+5+100}{6}$, but it is not true, it fails the above example.



Any help or hint will be appriciated, thanks.







optimization average median






share|cite|improve this question















share|cite|improve this question













share|cite|improve this question




share|cite|improve this question








edited 18 hours ago







Abdenaceur Lichiheb

















asked yesterday









Abdenaceur LichihebAbdenaceur Lichiheb

1436




1436












  • $begingroup$
    The answer is wrong. For example, $100$ is closer to each element than $104$ is.
    $endgroup$
    – Harnak
    yesterday






  • 2




    $begingroup$
    math.stackexchange.com/questions/113270/…
    $endgroup$
    – Matti P.
    yesterday










  • $begingroup$
    @Harnak Sorry but I don't quite understand your statement. $104$ is the total distance $sum_{x in S} |x - n|$ where $n$ is the desired number and $S = {1, 2, 5, 5, 7, 100}$. How do you achieve $100$? See my proof.
    $endgroup$
    – L. F.
    yesterday










  • $begingroup$
    @L.F., I was just answering to the OP before editing. He stated that the solution was 104, but I thought he referred to the minimizer and not to the distance. So, that's why I provided an example of why 104 couldn't be a minimizer. I think I just misinterpreted what he meant.
    $endgroup$
    – Harnak
    yesterday










  • $begingroup$
    @Harnak Oh, never mind. Everybody makes mistakes :)
    $endgroup$
    – L. F.
    yesterday


















  • $begingroup$
    The answer is wrong. For example, $100$ is closer to each element than $104$ is.
    $endgroup$
    – Harnak
    yesterday






  • 2




    $begingroup$
    math.stackexchange.com/questions/113270/…
    $endgroup$
    – Matti P.
    yesterday










  • $begingroup$
    @Harnak Sorry but I don't quite understand your statement. $104$ is the total distance $sum_{x in S} |x - n|$ where $n$ is the desired number and $S = {1, 2, 5, 5, 7, 100}$. How do you achieve $100$? See my proof.
    $endgroup$
    – L. F.
    yesterday










  • $begingroup$
    @L.F., I was just answering to the OP before editing. He stated that the solution was 104, but I thought he referred to the minimizer and not to the distance. So, that's why I provided an example of why 104 couldn't be a minimizer. I think I just misinterpreted what he meant.
    $endgroup$
    – Harnak
    yesterday










  • $begingroup$
    @Harnak Oh, never mind. Everybody makes mistakes :)
    $endgroup$
    – L. F.
    yesterday
















$begingroup$
The answer is wrong. For example, $100$ is closer to each element than $104$ is.
$endgroup$
– Harnak
yesterday




$begingroup$
The answer is wrong. For example, $100$ is closer to each element than $104$ is.
$endgroup$
– Harnak
yesterday




2




2




$begingroup$
math.stackexchange.com/questions/113270/…
$endgroup$
– Matti P.
yesterday




$begingroup$
math.stackexchange.com/questions/113270/…
$endgroup$
– Matti P.
yesterday












$begingroup$
@Harnak Sorry but I don't quite understand your statement. $104$ is the total distance $sum_{x in S} |x - n|$ where $n$ is the desired number and $S = {1, 2, 5, 5, 7, 100}$. How do you achieve $100$? See my proof.
$endgroup$
– L. F.
yesterday




$begingroup$
@Harnak Sorry but I don't quite understand your statement. $104$ is the total distance $sum_{x in S} |x - n|$ where $n$ is the desired number and $S = {1, 2, 5, 5, 7, 100}$. How do you achieve $100$? See my proof.
$endgroup$
– L. F.
yesterday












$begingroup$
@L.F., I was just answering to the OP before editing. He stated that the solution was 104, but I thought he referred to the minimizer and not to the distance. So, that's why I provided an example of why 104 couldn't be a minimizer. I think I just misinterpreted what he meant.
$endgroup$
– Harnak
yesterday




$begingroup$
@L.F., I was just answering to the OP before editing. He stated that the solution was 104, but I thought he referred to the minimizer and not to the distance. So, that's why I provided an example of why 104 couldn't be a minimizer. I think I just misinterpreted what he meant.
$endgroup$
– Harnak
yesterday












$begingroup$
@Harnak Oh, never mind. Everybody makes mistakes :)
$endgroup$
– L. F.
yesterday




$begingroup$
@Harnak Oh, never mind. Everybody makes mistakes :)
$endgroup$
– L. F.
yesterday










2 Answers
2






active

oldest

votes


















9












$begingroup$

You are looking to minimize $$sum_{y in A} |y - x|$$
with respect to $x$ where $A$ is your set.



It can be proved that any median minimizes this problem. In your case, the only median is $5$, so that's the result.






share|cite|improve this answer











$endgroup$









  • 9




    $begingroup$
    I upvoted your answer because it's correct, but I'd like to make a suggestion about the phrase "this is a well known result": it appears to serve the purpose of making you look well-read, and the OP look ignorant, which might both be true, but does saying it help? Probably not. If you'd included a reference, that'd be great -- OP could learn something. Most likely, this exercise is leading up to the very result you cited, and explaining the computation in this case would be more educational than citing the very thing that can be proved in general from similar computations.
    $endgroup$
    – John Hughes
    yesterday










  • $begingroup$
    You're right. However, that was not my intent. I'll edit this. Thanks for the suggestion :)
    $endgroup$
    – Harnak
    yesterday



















4












$begingroup$

First sort your [multi]set: ${1, 2, 5, 5, 7, 100}$. The number you want is $5$. The sum is $4 + 3 + 0 + 0 + 2 + 95 = 104.$



Proof: suppose you have another number $n neq 5$.



Note that for any number $x$, $|x - a| + |x - b| le |a - b|$.
Hence, it must hold that the sum of its distances to the two $5$s, i.e. $$|n - 5| + |n - 5| ge |5 - 5| + |5 - 5| = 0.$$ Similarly, $$|n - 2| + |n - 7| ge |5 - 2| + |5 - 7| = 5,$$
$$|n - 1| + |n - 100| ge |5 - 1| + |5 - 100| = 99.$$



You can't have the total distance any lower.
Q.E.D.



In general, first sort your set, then any number between (including) the middle two numbers will do. For example, for set ${1, 2, 3, 4, 5, 6}$, any $x$ such that $3 le x le 4$ does.






share|cite|improve this answer











$endgroup$













    Your Answer





    StackExchange.ifUsing("editor", function () {
    return StackExchange.using("mathjaxEditing", function () {
    StackExchange.MarkdownEditor.creationCallbacks.add(function (editor, postfix) {
    StackExchange.mathjaxEditing.prepareWmdForMathJax(editor, postfix, [["$", "$"], ["\\(","\\)"]]);
    });
    });
    }, "mathjax-editing");

    StackExchange.ready(function() {
    var channelOptions = {
    tags: "".split(" "),
    id: "69"
    };
    initTagRenderer("".split(" "), "".split(" "), channelOptions);

    StackExchange.using("externalEditor", function() {
    // Have to fire editor after snippets, if snippets enabled
    if (StackExchange.settings.snippets.snippetsEnabled) {
    StackExchange.using("snippets", function() {
    createEditor();
    });
    }
    else {
    createEditor();
    }
    });

    function createEditor() {
    StackExchange.prepareEditor({
    heartbeatType: 'answer',
    autoActivateHeartbeat: false,
    convertImagesToLinks: true,
    noModals: true,
    showLowRepImageUploadWarning: true,
    reputationToPostImages: 10,
    bindNavPrevention: true,
    postfix: "",
    imageUploader: {
    brandingHtml: "Powered by u003ca class="icon-imgur-white" href="https://imgur.com/"u003eu003c/au003e",
    contentPolicyHtml: "User contributions licensed under u003ca href="https://creativecommons.org/licenses/by-sa/3.0/"u003ecc by-sa 3.0 with attribution requiredu003c/au003e u003ca href="https://stackoverflow.com/legal/content-policy"u003e(content policy)u003c/au003e",
    allowUrls: true
    },
    noCode: true, onDemand: true,
    discardSelector: ".discard-answer"
    ,immediatelyShowMarkdownHelp:true
    });


    }
    });














    draft saved

    draft discarded


















    StackExchange.ready(
    function () {
    StackExchange.openid.initPostLogin('.new-post-login', 'https%3a%2f%2fmath.stackexchange.com%2fquestions%2f3092033%2ffind-a-number-having-minimum-sum-of-distances-between-a-set-of-numbers%23new-answer', 'question_page');
    }
    );

    Post as a guest















    Required, but never shown

























    2 Answers
    2






    active

    oldest

    votes








    2 Answers
    2






    active

    oldest

    votes









    active

    oldest

    votes






    active

    oldest

    votes









    9












    $begingroup$

    You are looking to minimize $$sum_{y in A} |y - x|$$
    with respect to $x$ where $A$ is your set.



    It can be proved that any median minimizes this problem. In your case, the only median is $5$, so that's the result.






    share|cite|improve this answer











    $endgroup$









    • 9




      $begingroup$
      I upvoted your answer because it's correct, but I'd like to make a suggestion about the phrase "this is a well known result": it appears to serve the purpose of making you look well-read, and the OP look ignorant, which might both be true, but does saying it help? Probably not. If you'd included a reference, that'd be great -- OP could learn something. Most likely, this exercise is leading up to the very result you cited, and explaining the computation in this case would be more educational than citing the very thing that can be proved in general from similar computations.
      $endgroup$
      – John Hughes
      yesterday










    • $begingroup$
      You're right. However, that was not my intent. I'll edit this. Thanks for the suggestion :)
      $endgroup$
      – Harnak
      yesterday
















    9












    $begingroup$

    You are looking to minimize $$sum_{y in A} |y - x|$$
    with respect to $x$ where $A$ is your set.



    It can be proved that any median minimizes this problem. In your case, the only median is $5$, so that's the result.






    share|cite|improve this answer











    $endgroup$









    • 9




      $begingroup$
      I upvoted your answer because it's correct, but I'd like to make a suggestion about the phrase "this is a well known result": it appears to serve the purpose of making you look well-read, and the OP look ignorant, which might both be true, but does saying it help? Probably not. If you'd included a reference, that'd be great -- OP could learn something. Most likely, this exercise is leading up to the very result you cited, and explaining the computation in this case would be more educational than citing the very thing that can be proved in general from similar computations.
      $endgroup$
      – John Hughes
      yesterday










    • $begingroup$
      You're right. However, that was not my intent. I'll edit this. Thanks for the suggestion :)
      $endgroup$
      – Harnak
      yesterday














    9












    9








    9





    $begingroup$

    You are looking to minimize $$sum_{y in A} |y - x|$$
    with respect to $x$ where $A$ is your set.



    It can be proved that any median minimizes this problem. In your case, the only median is $5$, so that's the result.






    share|cite|improve this answer











    $endgroup$



    You are looking to minimize $$sum_{y in A} |y - x|$$
    with respect to $x$ where $A$ is your set.



    It can be proved that any median minimizes this problem. In your case, the only median is $5$, so that's the result.







    share|cite|improve this answer














    share|cite|improve this answer



    share|cite|improve this answer








    edited yesterday

























    answered yesterday









    HarnakHarnak

    1,082412




    1,082412








    • 9




      $begingroup$
      I upvoted your answer because it's correct, but I'd like to make a suggestion about the phrase "this is a well known result": it appears to serve the purpose of making you look well-read, and the OP look ignorant, which might both be true, but does saying it help? Probably not. If you'd included a reference, that'd be great -- OP could learn something. Most likely, this exercise is leading up to the very result you cited, and explaining the computation in this case would be more educational than citing the very thing that can be proved in general from similar computations.
      $endgroup$
      – John Hughes
      yesterday










    • $begingroup$
      You're right. However, that was not my intent. I'll edit this. Thanks for the suggestion :)
      $endgroup$
      – Harnak
      yesterday














    • 9




      $begingroup$
      I upvoted your answer because it's correct, but I'd like to make a suggestion about the phrase "this is a well known result": it appears to serve the purpose of making you look well-read, and the OP look ignorant, which might both be true, but does saying it help? Probably not. If you'd included a reference, that'd be great -- OP could learn something. Most likely, this exercise is leading up to the very result you cited, and explaining the computation in this case would be more educational than citing the very thing that can be proved in general from similar computations.
      $endgroup$
      – John Hughes
      yesterday










    • $begingroup$
      You're right. However, that was not my intent. I'll edit this. Thanks for the suggestion :)
      $endgroup$
      – Harnak
      yesterday








    9




    9




    $begingroup$
    I upvoted your answer because it's correct, but I'd like to make a suggestion about the phrase "this is a well known result": it appears to serve the purpose of making you look well-read, and the OP look ignorant, which might both be true, but does saying it help? Probably not. If you'd included a reference, that'd be great -- OP could learn something. Most likely, this exercise is leading up to the very result you cited, and explaining the computation in this case would be more educational than citing the very thing that can be proved in general from similar computations.
    $endgroup$
    – John Hughes
    yesterday




    $begingroup$
    I upvoted your answer because it's correct, but I'd like to make a suggestion about the phrase "this is a well known result": it appears to serve the purpose of making you look well-read, and the OP look ignorant, which might both be true, but does saying it help? Probably not. If you'd included a reference, that'd be great -- OP could learn something. Most likely, this exercise is leading up to the very result you cited, and explaining the computation in this case would be more educational than citing the very thing that can be proved in general from similar computations.
    $endgroup$
    – John Hughes
    yesterday












    $begingroup$
    You're right. However, that was not my intent. I'll edit this. Thanks for the suggestion :)
    $endgroup$
    – Harnak
    yesterday




    $begingroup$
    You're right. However, that was not my intent. I'll edit this. Thanks for the suggestion :)
    $endgroup$
    – Harnak
    yesterday











    4












    $begingroup$

    First sort your [multi]set: ${1, 2, 5, 5, 7, 100}$. The number you want is $5$. The sum is $4 + 3 + 0 + 0 + 2 + 95 = 104.$



    Proof: suppose you have another number $n neq 5$.



    Note that for any number $x$, $|x - a| + |x - b| le |a - b|$.
    Hence, it must hold that the sum of its distances to the two $5$s, i.e. $$|n - 5| + |n - 5| ge |5 - 5| + |5 - 5| = 0.$$ Similarly, $$|n - 2| + |n - 7| ge |5 - 2| + |5 - 7| = 5,$$
    $$|n - 1| + |n - 100| ge |5 - 1| + |5 - 100| = 99.$$



    You can't have the total distance any lower.
    Q.E.D.



    In general, first sort your set, then any number between (including) the middle two numbers will do. For example, for set ${1, 2, 3, 4, 5, 6}$, any $x$ such that $3 le x le 4$ does.






    share|cite|improve this answer











    $endgroup$


















      4












      $begingroup$

      First sort your [multi]set: ${1, 2, 5, 5, 7, 100}$. The number you want is $5$. The sum is $4 + 3 + 0 + 0 + 2 + 95 = 104.$



      Proof: suppose you have another number $n neq 5$.



      Note that for any number $x$, $|x - a| + |x - b| le |a - b|$.
      Hence, it must hold that the sum of its distances to the two $5$s, i.e. $$|n - 5| + |n - 5| ge |5 - 5| + |5 - 5| = 0.$$ Similarly, $$|n - 2| + |n - 7| ge |5 - 2| + |5 - 7| = 5,$$
      $$|n - 1| + |n - 100| ge |5 - 1| + |5 - 100| = 99.$$



      You can't have the total distance any lower.
      Q.E.D.



      In general, first sort your set, then any number between (including) the middle two numbers will do. For example, for set ${1, 2, 3, 4, 5, 6}$, any $x$ such that $3 le x le 4$ does.






      share|cite|improve this answer











      $endgroup$
















        4












        4








        4





        $begingroup$

        First sort your [multi]set: ${1, 2, 5, 5, 7, 100}$. The number you want is $5$. The sum is $4 + 3 + 0 + 0 + 2 + 95 = 104.$



        Proof: suppose you have another number $n neq 5$.



        Note that for any number $x$, $|x - a| + |x - b| le |a - b|$.
        Hence, it must hold that the sum of its distances to the two $5$s, i.e. $$|n - 5| + |n - 5| ge |5 - 5| + |5 - 5| = 0.$$ Similarly, $$|n - 2| + |n - 7| ge |5 - 2| + |5 - 7| = 5,$$
        $$|n - 1| + |n - 100| ge |5 - 1| + |5 - 100| = 99.$$



        You can't have the total distance any lower.
        Q.E.D.



        In general, first sort your set, then any number between (including) the middle two numbers will do. For example, for set ${1, 2, 3, 4, 5, 6}$, any $x$ such that $3 le x le 4$ does.






        share|cite|improve this answer











        $endgroup$



        First sort your [multi]set: ${1, 2, 5, 5, 7, 100}$. The number you want is $5$. The sum is $4 + 3 + 0 + 0 + 2 + 95 = 104.$



        Proof: suppose you have another number $n neq 5$.



        Note that for any number $x$, $|x - a| + |x - b| le |a - b|$.
        Hence, it must hold that the sum of its distances to the two $5$s, i.e. $$|n - 5| + |n - 5| ge |5 - 5| + |5 - 5| = 0.$$ Similarly, $$|n - 2| + |n - 7| ge |5 - 2| + |5 - 7| = 5,$$
        $$|n - 1| + |n - 100| ge |5 - 1| + |5 - 100| = 99.$$



        You can't have the total distance any lower.
        Q.E.D.



        In general, first sort your set, then any number between (including) the middle two numbers will do. For example, for set ${1, 2, 3, 4, 5, 6}$, any $x$ such that $3 le x le 4$ does.







        share|cite|improve this answer














        share|cite|improve this answer



        share|cite|improve this answer








        edited yesterday

























        answered yesterday









        L. F.L. F.

        1737




        1737






























            draft saved

            draft discarded




















































            Thanks for contributing an answer to Mathematics Stack Exchange!


            • Please be sure to answer the question. Provide details and share your research!

            But avoid



            • Asking for help, clarification, or responding to other answers.

            • Making statements based on opinion; back them up with references or personal experience.


            Use MathJax to format equations. MathJax reference.


            To learn more, see our tips on writing great answers.




            draft saved


            draft discarded














            StackExchange.ready(
            function () {
            StackExchange.openid.initPostLogin('.new-post-login', 'https%3a%2f%2fmath.stackexchange.com%2fquestions%2f3092033%2ffind-a-number-having-minimum-sum-of-distances-between-a-set-of-numbers%23new-answer', 'question_page');
            }
            );

            Post as a guest















            Required, but never shown





















































            Required, but never shown














            Required, but never shown












            Required, but never shown







            Required, but never shown

































            Required, but never shown














            Required, but never shown












            Required, but never shown







            Required, but never shown







            Popular posts from this blog

            If I really need a card on my start hand, how many mulligans make sense? [duplicate]

            Alcedinidae

            Can an atomic nucleus contain both particles and antiparticles? [duplicate]